Difference between revisions of "2023 AMC 8 Problems/Problem 10"

(Solution 1)
Line 1: Line 1:
==Solution 1==  
+
==Problem==
  
Harold made a plum pie. We can distribute into different fractions of pie that the animals ate. First Harold ate <math>\frac{1}{4}</math> of the pie and the remains are <math>\frac{3}{4}</math> of the pie. From this a moose comes and eats <math>\frac{3}{4} * \frac{1}{3}</math> and what remains is <math>\frac{3}{4} - \frac{1}{4} = \frac{1}{2}</math>. From this a porcupine comes and eat <math>\frac{1}{3} * \frac{1}{2} = \frac{1}{6}</math> and our final answer of what remains is <math>\frac{1}{2} - \frac{1}{6} =  \boxed{\text{(D)}\frac{1}{3}}</math>.  
+
Harold made a plum pie to take on a picnic. He was able to eat only <math>\frac{1}{4}</math> of the pie, and he left the rest for his friends. A moose came by and ate <math>\frac{1}{3}</math> of what Harold left behind. After that, a porcupine ate <math>\frac{1}{3}</math> of what the moose left behind. How much of the original pie still remained after the porcupine left?
  
 +
<math>\textbf{(A)}\ \frac{1}{12} \qquad \textbf{(B)}\ \frac{1}{6} \qquad \textbf{(C)}\ \frac{1}{4} \qquad \textbf{(D)}\ \frac{1}{3} \qquad \textbf{(E)}\ \frac{5}{12}</math>
  
~apex304, SohumUttamchandani, wuwang2002, TaeKim, Cxrupptedpat, lpieleanu
+
==Solution==
 +
 
 +
Note that:
 +
 
 +
 
 +
 
 +
Alternatively, we can condense the solution above into the following equation: <cmath>\left(1-\frac14\right)\left(1-\frac13\right)\left(1-\frac13\right) = \frac34\cdot\frac23\cdot\frac23 = \frac13.</cmath>
 +
 
 +
~apex304, SohumUttamchandani, wuwang2002, TaeKim, Cxrupptedpat, lpieleanu, MRENTHUSIASM
 +
 
 +
==See Also==
 +
{{AMC8 box|year=2023|num-b=9|num-a=11}}
 +
{{MAA Notice}}

Revision as of 00:25, 25 January 2023

Problem

Harold made a plum pie to take on a picnic. He was able to eat only $\frac{1}{4}$ of the pie, and he left the rest for his friends. A moose came by and ate $\frac{1}{3}$ of what Harold left behind. After that, a porcupine ate $\frac{1}{3}$ of what the moose left behind. How much of the original pie still remained after the porcupine left?

$\textbf{(A)}\ \frac{1}{12} \qquad \textbf{(B)}\ \frac{1}{6} \qquad \textbf{(C)}\ \frac{1}{4} \qquad \textbf{(D)}\ \frac{1}{3} \qquad \textbf{(E)}\ \frac{5}{12}$

Solution

Note that:


Alternatively, we can condense the solution above into the following equation: \[\left(1-\frac14\right)\left(1-\frac13\right)\left(1-\frac13\right) = \frac34\cdot\frac23\cdot\frac23 = \frac13.\]

~apex304, SohumUttamchandani, wuwang2002, TaeKim, Cxrupptedpat, lpieleanu, MRENTHUSIASM

See Also

2023 AMC 8 (ProblemsAnswer KeyResources)
Preceded by
Problem 9
Followed by
Problem 11
1 2 3 4 5 6 7 8 9 10 11 12 13 14 15 16 17 18 19 20 21 22 23 24 25
All AJHSME/AMC 8 Problems and Solutions

The problems on this page are copyrighted by the Mathematical Association of America's American Mathematics Competitions. AMC logo.png